mehr Fragen

This commit is contained in:
WieErWill 2022-02-12 10:46:06 +01:00
parent 94c50ee69e
commit 804f7e8719
2 changed files with 133 additions and 2 deletions

View File

@ -102,6 +102,11 @@
Dazu muss $l\in V$ und $r\in \sum V\cup {\epsilon}$.
\end{solution}
\part Die Menge $Reg(\sum)$ der regulären Ausdrücke über dem Alphabet ist...
\begin{solution}
\end{solution}
\part Ein NFA ist ein Tupel $M=(...)$
\begin{solution}
ein nichtdeterministischer endlicher Automat $M$ ist ein 5-Tupel $M=(Z,\sum,S,\delta,E)$ mit
@ -114,11 +119,21 @@
\end{itemize}
\end{solution}
\part Die von einem NFA $M=(Z,\sum,S,\delta, E)$ akzeptierte Sprache ist $L(M)=...$ (ohne Definition der Mehr-Schritt Übergangsfunktion $\delta$)
\begin{solution}
\end{solution}
\part Die von einem PDA $M=(Z,\sum, \Gamma, \delta, z_0, \#)$ akzeptierten Sprache ist $L(M)=...$
\begin{solution}
$L(M)=\{x\in\sum^* | \text{ es gibt } z\in Z \text{ mit } (z_0, x, \#) [...] ^*(z,\epsilon, \epsilon)\}$
\end{solution}
\part Sei $L$ eine Sprache. Für $x,y\in\sum^*$ gilt $xE_L y$ genau dann, wenn ... ($R_L$ ist die Myhill-Nerode-Äquivalenz zu L)
\begin{solution}
\end{solution}
\part Sei $M=(Z,\sum,z_0,\delta, E)$ ein DFA. Die Zustände $z,z'\in Z$ heißen erkennungsäquivalent, wenn
\begin{solution}
Zwei Zustände $z,z'\in Z$ heißen erkennungsäquivalent ($z\equiv z'$) wenn für jedes Wort $w\in \sum^*$ gilt: $\hat{\sigma}(z,w)\in E \leftrightarrow \hat{\sigma}(z',w)\in E$.
@ -136,6 +151,26 @@
\end{enumerate}
\end{solution}
\part Die Klasse der regulären Sprachen ist unter anderem abgeschlossen unter folgenden drei Operationen:
\begin{solution}
\end{solution}
\part Sei $\sum$ ein Alphabet. Die Anzahl der Grammatiken über $\sum$ ist ... und die Anzahl der Sprachen über $\sum$ ist ... .
\begin{solution}
\end{solution}
\part Unter anderem sind folgende (mind. drei) Probleme für kontextfreie Sprachen entscheidbar:
\begin{solution}
\end{solution}
\part Die Klasse der Kontextfreien Sprachen ist abgeschlossen unter den Operationen 1)... und 2)... . Sie ist aber nicht abgeschlossen unter 3)... und 4)... .
\begin{solution}
\end{solution}
\part Der Satz von Myhill-Nerode besagt,...
\begin{solution}
Sei L eine Sprache. L ist regulär $\leftrightarrow index(R_L)< \infty$ (d.h. nur wenn die Myhill-Nerode-Äquivalenz endliche Klassen hat).
@ -157,6 +192,26 @@
\question Konstruktionen der Automatentheorie
\begin{parts}
\part Betrachte den folgenden NFA X. Berechne einen DFA Y mit $L(X)=L(Y)$.
\begin{center}
\begin{tikzpicture}[node distance = 3cm, on grid, auto]
\node (q1) [state, initial, initial text = {}] {1};
\node (q2) [state, above right = of q1] {2};
\node (q3) [state, accepting, below right = of q2] {3};
\path [-stealth, thick]
(q1) edge node {a} (q2)
(q1) edge node {b} (q3)
(q1) edge [loop above] node {a}()
(q2) edge node {a} (q3)
(q2) edge [loop above] node {b}()
(q3) edge [bend left] node {a} (q2);
\end{tikzpicture}
\end{center}
\begin{solution}
\end{solution}
\part Betrachte den folgenden NFA X. Berechne einen DFA Y mit $L(X)=L(Y)$.
\begin{center}
\begin{tikzpicture}[node distance = 3cm, on grid, auto]
@ -175,10 +230,10 @@
(q2) edge [loop above] node {b}();
\end{tikzpicture}
\end{center}
\begin{solution}
\end{solution}
\part Betrachte den folgenden DFA X (Bild wird noch erstellt). Berechne den minimalen DFA Y mit $L(X)=L(Y)$.
\part Betrachte den folgenden DFA X. Berechne den minimalen DFA Y mit $L(X)=L(Y)$.
\begin{center}
\begin{tikzpicture}[node distance = 3cm, on grid, auto]
\node (q1) [state, initial, initial text = {}] {1};
@ -204,6 +259,35 @@
\end{center}
\begin{solution}
\end{solution}
\part Betrachte den folgenden DFA X. Berechne den minimalen DFA Y mit $L(X)=L(Y)$.
\begin{center}
\begin{tikzpicture}[node distance = 3cm, on grid, auto]
\node (q1) [state, accepting, initial, initial text = {}] {1};
\node (q2) [state, accepting, right = of q1] {2};
\node (q3) [state, right = of q2] {3};
\node (q4) [state, accepting, below = of q1] {4};
\node (q5) [state, right = of q4] {5};
\node (q6) [state, accepting, right = of q5] {6};
\path [-stealth, thick]
(q1) edge node {b} (q2)
(q1) edge node {a} (q5)
(q2) edge [loop above] node {b}()
(q2) edge node {a} (q3)
(q3) edge node {a} (q5)
(q3) edge node {b} (q6)
(q4) edge node {a} (q1)
(q4) edge node {b} (q5)
(q5) edge [bend left] node {a} (q3)
(q5) edge [bend left] node {b} (q4)
(q6) edge [bend left] node {a} (q2)
(q6) edge node {b} (q5);
\end{tikzpicture}
\end{center}
\begin{solution}
\end{solution}
\end{parts}
\question Algorithmen für reguläre Sprachen. Sei $\sum=\{a,b,c\}$. Gebe einen Algorithmus an, der bei Eingabe eines NFA X entscheidet, ob alle Wörter $\omega\in L(X)$ ungerade Länge besitzen und $abc$ als Infix enthalten.
@ -241,19 +325,42 @@
\part Ein While Programm ist von der Form...
\begin{solution}
\end{solution}
\part Eine Turingmaschine ist ein 7-Tupel $M=(Z,\sum,\Gamma,\delta,z_0,\Box, E)$, wobei...
\begin{solution}
\end{solution}
\part Die von einer Turingmaschine $M$ akzeptierte Sprache ist $L(M)=...$
\begin{solution}
\end{solution}
\part Seien $A\supseteq \sum^*$ und B$\supseteq \Gamma^*$. Eine Reduktion von A auf B ist ...
\begin{solution}
\end{solution}
\part Eine Sprache $L$ heißt rekursiv aufzählbar, falls ...
\begin{solution}
\end{solution}
\part Sei $f:N\rightarrow N$ eine monotone Funktion. Die Klasse $TIME(f)$ besteht aus allen Sprachen L, für die es eine Turingmaschine $M$ gibt mit ...
\begin{solution}
\end{solution}
\end{parts}
\question Sätze der Berechnbarkeitstheorie: Vervollständige die folgenden Aussagen
\begin{parts}
\part Zu jeder Mehrband-Turingmaschine $M$ gibt es ...
\begin{solution}
\end{solution}
\part Sei $f:N^k\rightarrow\mathbb{N}$ eine Funktion für ein $k\in\mathbb{N}$. Die folgenden Aussagen sind äquivalent: 1) $f$ ist Turing-berechenbar, 2)..., 3)..., 4)...
\begin{solution}
\end{solution}
\part Sei $L\subseteq \sum^*$ eine Sprache. Sind $L$ und $\sum^*\backslash L$ semi-entscheidbar, dann...
\begin{solution}
\end{solution}
\part Der Satz von Rice lautet...
\begin{solution}
dass es unmöglich ist, eine beliebige nicht-triviale Eigenschaft der erzeugten Funktion einer Turing-Maschine (oder eines Algorithmus in einem anderen Berechenbarkeitsmodell) algorithmisch zu entscheiden.
@ -269,6 +376,15 @@
\part Gebe ein Loop-Programm an, das die Funktion $n\rightarrow n^2-n$ berechnet
\begin{solution}
\end{solution}
\part Gebe ein Loop Programm an, das die Funktion $f:\mathbb{N}\rightarrow \mathbb{N}$ mit $f(n_1,n_2)=2n_1n_2$ berechnet. Verwende nur elementare Anweisungen und keine Abkürzungen.
\begin{solution}
\end{solution}
\part Gebe ein GoTo Programm an, das die Funktion $g:\mathbb{N}\rightarrow\mathbb{N}$ mit $g(n_1,n_2)=|n_1-n_2|$ berechnet. Verwende nur elementare Anweisungen und keine Abkürzungen.
\begin{solution}
\end{solution}
\part Gebe eine deterministische Turingmaschine $M$ für das Eingabealphabet $\{0,1\}$ an, das folgende Funktion berechnet: Für Eingabe $a_1a_2...a_{n-1}a_n$ berechnet M die Ausgabe $a_na_1...a_{n-1}$ (letzte Symbol der Eingabe an erste Stelle).
\begin{solution}
\end{solution}
@ -304,6 +420,10 @@
\end{solution}
\end{parts}
\question Unentscheidbare Probleme: Gebe (mind vier) unterscheidbare Probleme an (als Menge oder als Eingabe-Frage-Paar).
\begin{solution}
\end{solution}
\question NP-vollständiges Problem: Gebe (mind. zwei) NP-vollständige Probleme an (als Menge oder Eingabe-Frage-Paar).
\begin{solution}
@ -315,5 +435,16 @@
\end{solution}
\question Polynomialzeitreduktion: Betrachte das Problem 4C, also die Menge der ungerichteten Graphen die sich mit vier Farben färben lassen.
\begin{parts}
\part Gebe eine Polynomialzeitreduktion von 3C auf 4C an.
\begin{solution}
\end{solution}
\part Zeige, dass wenn $4C\in P$, dann gilt $P=NP$.
\begin{solution}
\end{solution}
\end{parts}
\end{questions}
\end{document}